Đến nội dung

baopbc nội dung

Có 386 mục bởi baopbc (Tìm giới hạn từ 09-05-2020)



Sắp theo                Sắp xếp  

#631571 $$\frac{a+b+c}{3}\geq \sqrt[k]...

Đã gửi bởi baopbc on 06-05-2016 - 14:24 trong Bất đẳng thức - Cực trị

Cho $a,b,c>0$ thỏa mãn $abc=1$. Tìm các giá trị hằng số $k$ sao cho:

$$\frac{a+b+c}{3}\geq \sqrt[k]{\frac{a^2+b^2+c^2}{3}}$$




#623956 $$\frac{a^{2}b}{c} + \frac...

Đã gửi bởi baopbc on 01-04-2016 - 00:50 trong Bất đẳng thức và cực trị

Làm sao để có thể phân tích ảo diệu đến như vậy hở anh ?  :wacko:

Mình nghĩ có thể anh ấy làm ngược lại, tức là từ vế sau khai triển ra vế đầu! :)

Hoắc cũng có thể có cách làm ảo diệu để dẫn đến như trên. Cách này mình cũng từng thấy anh Huyện dùng rồi! :)




#634096 $$x_{n^2+1}=x_{n}.x_{n+1};\foral...

Đã gửi bởi baopbc on 19-05-2016 - 17:57 trong Dãy số - Giới hạn

Nếu vậy thì xác định $x_4$ ra làm sao nhỉ? :mellow:

$4$ đâu có dạng $n^2+1$? :mellow:




#598344 $\boxed{Topic}$Tìm các chữ số tận cùng của một tích, một lũy t...

Đã gửi bởi baopbc on 14-11-2015 - 22:18 trong Chuyên đề toán THCS

Tìm 2 chữ số tận cùng trong phần cơ sở của:

[$\sqrt{2}+^{\sqrt{3}}$]$^{2015}$




#607865 $\Delta AMN$ cân

Đã gửi bởi baopbc on 08-01-2016 - 08:12 trong Hình học

Câu a dễ thôi, cộng góc là được./




#634904 $\frac {n^m-m^n}{m+n} \in \mathbb...

Đã gửi bởi baopbc on 23-05-2016 - 10:19 trong Số học

Xem tại đây: http://diendantoanho...-olympiad-2016/




#608232 $\frac{1}{a^{2}}+\frac{1...

Đã gửi bởi baopbc on 09-01-2016 - 22:07 trong Bất đẳng thức và cực trị

$\frac{1}{a^{2}}+\frac{1}{b^{2}}+\frac{1}{c^{2}}+ab+bc+ca+ab+bc+ca\geq 9\sqrt[9]{\frac{1}{a^{2}b^{2}c^{2}}.(abc)^{4}}=9\sqrt[9]{(abc)^{2}}$  mà bạn

Thành thật xin lỗi, mình bị nhầm./




#608079 $\frac{1}{a^{2}}+\frac{1...

Đã gửi bởi baopbc on 08-01-2016 - 22:08 trong Bất đẳng thức và cực trị

Bài này giải vậy: cộng hai vế với 2ab+2bc+2ac thì VP=9

VT dùng AM-GM là được./




#608229 $\frac{1}{a^{2}}+\frac{1...

Đã gửi bởi baopbc on 09-01-2016 - 22:01 trong Bất đẳng thức và cực trị

Cụ thể là vận dụng thế nào bạn?

$\sum \frac{1}{a^{2}}+2\sum ab\geq 9\sqrt[9]{\frac{1}{(abc)^{2}}.(abc)^{2}}=9$




#620922 $\frac{1}{R}=\frac{1}{AP...

Đã gửi bởi baopbc on 18-03-2016 - 17:49 trong Hình học

Bài này khá hay, mô hình nó khá giống với bài hình học ngày 2 Vietnam TST 2015! Ngoài tính chất trên nó còn có nhiều tính chất khác rất thú vị!

Lời giải của mình: Để giải quyết bài toán, ta cần có bổ đề sau: Cho $\triangle ABC$ cân nội tiếp $(O)$. $D$ là điểm bất kì trên cung $BC$ không chứa $A$. Khi đó: 

$\frac {AD}{AC}=\frac {BD+DC}{BC}$.

Chứng minh: Trên $BD$ lấy $E$ sao cho $DE=DC$. Do $\triangle CDE$ cân tại $D$ nên ta có biến đổi góc: $\angle CED=\angle CDB/2= \angle ADC $. Mặt khác lại có:

$\angle CAD=\angle CBD \Rightarrow \triangle CAD \sim \triangle CBE$

$\Rightarrow \frac {AD}{AC}=\frac {BE}{BC}=\frac {BD+DC}{BC}$. Bổ đề được chứng minh.

Quay lại với bài toán: Nhận xét rằng: $\angle BAP=\angle ACP, \angle CAP=\angle ABP$ nên $(APB)$ tiếp xúc $AC$ và đi qua $B$; $(APC)$ tiếp xúc $AB$ và đi qua $C$. Dễ thấy $\triangle APB \sim \triangle CPA (g.g)$

Do $M,N$ lần lượt là các tâm nội tiếp $\triangle APB$ và $\triangle APC$ nên dễ dàng suy ra $\triangle MPN \sim \triangle APC ;O$ thuộc $AP$ ($O$ là tâm đường tròn ngoại tiếp tam giác $AMN$); $M,N,O,P$ đồng viên.

$\Rightarrow \frac {AP}{AB}= \frac {PC}{AC} \Rightarrow \frac {1}{R}=\frac {1}{AP}+\frac {1}{AB}+\frac {1}{AC} \Leftrightarrow \frac {AP}{R}=1+\frac {AP}{AB}+\frac {AP}{AC}=1+\frac {PA+PC}{AC}=\frac {MP+PM}{MN}+1 \Leftrightarrow \frac {OP}{R}=\frac {MP+PM}{MN}$

Đẳng thức cuối đúng theo bổ đề nên ta có điều phải chứng minh! :)

Hình gửi kèm

  • Post 7.jpg
  • Post 8.jpg



#632330 $\frac{1!2!+2!3!+...+n!(n+1)!}...

Đã gửi bởi baopbc on 10-05-2016 - 20:57 trong Bất đẳng thức và cực trị

Bài này không biết có uẩn khúc gì không nhỉ

Lời giải.$n=1;2$. Dễ thấy bất đẳng thức đúng.

$n\geq 3\Longrightarrow \frac {n+2}{\sqrt{n+1}}\geq 2$.

Bất đẳng thức cần chứng minh tương đương với:

$\frac{(1!2!+2!3!+...+n!(n+1)!)^n}{n^n(1!)^2.(2!)^2...(n!)^2}\geq 2^n\sqrt{n!}$

Áp dụng bất đẳng thức C-S ta được:

$\frac{(1!2!+2!3!+...+n!(n+1)!)^n}{n^n.(1!)^2.(2!)^2...(n!)^2}\geq \frac{(n\sqrt[n]{1!(2!)^2(3!)^2...(n!)^2.(n+1)!})^n}{n^n.(1!)^2.(2!)^2...(n!)^2}=\frac {n^n.1!(2!)^2(3!)^2...(n!)^2.(n+1)!}{n^n.(1!)^2.(2!)^2...(n!)^2}=(n+1)!$

Mặt khác với $n=3$ thì $(n+1)!\geq 2^n\sqrt{n!}$ nên theo nguyên lý quy nạp ta có điều phải chứng minh.




#632277 $(x-y)f(x+y)-(x+y)f(x-y)=4xy(x^2-y^2)$

Đã gửi bởi baopbc on 10-05-2016 - 17:00 trong Phương trình hàm

Tìm tất cả các hàm $f:R\rightarrow R$ thỏa mãn: 

$(x-y)f(x+y)-(x+y)f(x-y)=4xy(x^2-y^2)$

Mình giải thế này! :)

Giả sử $f$ là hàm thỏa mãn yêu cầu bài toán. Thay $x=y=1$ vào $(1)$ ta được: $-2.f(0)=0\Longrightarrow f(0)=0$.

Sử dụng phép đặt $x+y=u,x-y=v$ thì $u.v=x^2-y^2$ ta được:

$v.f(u)+u.f(v)=u.v(u^2-v^2)$. Xét $2$ khả năng sau:

Khả năng $1:u.v\neq 0\Longrightarrow \frac {f(u)}{u}-\frac {f(v)}{v}=u^2-v^2\Longrightarrow \frac {f(u)}{u}-u^2=\frac {f(v)}{v}-v^2\forall u.v\neq 0;u,v\in \mathbb{R}$

Thay $v=1$ ta suy ra $\frac {f(u)}{u}-u^2=f(1)-1\Longrightarrow f(u)=(f(1)-1)u+u^3\forall u\neq 0$

Mặt khác kết hợp với $f(0)=0\Longrightarrow f(u)=(f(1)-1)u+u^3\forall u\in \mathbb{R}$ (thử lại thỏa mãn)

Khả năng $2:u.v=0\Longrightarrow f(u)=0\forall u\in \mathbb{R}$

Thử lại chỉ có hàm $f(x)=(f(1)-1)x+x^3$ thỏa mãn.

Kết luận: $\boxed{f(x)=(f(1)-1)x+x^3}$

P/s:

Mọi người xem lại hộ em với! :)




#633631 $(z + 1)f(x + y) = f(xf(z) + y) + f(yf(z) + x)$

Đã gửi bởi baopbc on 17-05-2016 - 12:41 trong Phương trình hàm

Bài này là APMO 2016 phải không nhỉ?

Mình vừa mở lại topic để mọi người thảo luận.  :D

Bài APMO 2016 là $f:\mathbb{R+}\rightarrow \mathbb{R+}$ chứ anh! Hoặc là anh Ego chế lại, hoặc là anh ấy viết sai đề rồi! :)




#614722 [Hình học] THCS tháng 12: Chứng minh $ST \parallel BC$

Đã gửi bởi baopbc on 13-02-2016 - 16:05 trong Thảo luận đề thi VMEO IV

[HIDE]Cách làm của em mang tính đại số hơi nhiều.Ai có cách giải thuần túy hình học thì post lên nhé [/H

Có nhiều bài toán gần như không thể xử lý bằng phương pháp thuần túy. Nếu có thì có thể là sử dụng một số kiến thức ở bậc THPT chẳng hạn>

Như bài tuần 3 tháng 1 tại Mỗi tuần một bài toán./

P/s: Nếu ai có lời giải thuần túy thì mình rất vui lòng được biết :)




#608863 [Hình học] THPT tháng 11: $UK$ đi qua điểm cố định khi $P, Q...

Đã gửi bởi baopbc on 13-01-2016 - 22:24 trong Thảo luận đề thi VMEO IV

. Mình giải bằng cách sử dụng trục đẳng phương (dĩ nhiên ngắn hơn của bạn :D )

Anh Hân đăng lời giải đi! :)




#614465 [Hình học] THPT tháng 12: Chứng minh $PA=PL$.

Đã gửi bởi baopbc on 12-02-2016 - 15:44 trong Thảo luận đề thi VMEO IV

Do tính chất đối xứng nên AP=AF, AP=AE$\Rightarrow$AF=AE$\Rightarrow$$\Delta AEF$ cân tại A

Từ đây suy ra AK đi qua trung điểm EF. Gọi T là trung điểm EF

Theo tính chất đường trung bình thì đường thẳng qua T và vuông góc với BC sẽ đi qua trung điểm AL và vuông góc với AL(*)

Vậy ta chỉ cần chứng minh PT vuông góc với BC.

X,Y,Z là chân đường vuông góc kẻ từ P tới BC,CA,AB.Ta có X,Y,Z thẳng hàng(theo tính chất của đường thẳng Simson) và YZ song song với EF

Dễ thấy $\Delta YPZ$ đồng dạng với $\Delta CPB$(g.g) (1) và $\angle XPY=\angle ACB$

Do AP là đối trung của tam giác ABC nên Pa là đối trung của tam giác BPC

Gọi M là trung điểm BC, ta có $\angle MPC=\angle APB=\angle ACB=\angle XPY$( theo tính chất đối trung)(2)

Từ (1)(2) ta suy ra X là trung điểm YZ$\Rightarrow$PX đi qua T$\Rightarrow$PT vuông góc với BC(**)

Từ (*)(**) ta suy ra tam giác PAL cân tại P$\Rightarrow$PA=PL(đpcm)




#617361 [Trường Xuân toán học miền nam 2016] Vietnam TST 2016 MOCK Test 2

Đã gửi bởi baopbc on 28-02-2016 - 12:45 trong Thi HSG cấp Tỉnh, Thành phố. Olympic 30-4. Đề thi và kiểm tra đội tuyển các cấp.

Hình như câu 5 đã có trong một số tài liệu rồi thì phải! Trên diễn đàn cũng có một topic về bài này rồi nhưng chưa ai giải cả!

Em xin gửi lại link: http://diendantoanho...-geq-1dfrac4rr/

Ở trong ấy anh Bui Ba Anh gõ sai đề, là 1 chứ không phải 1/2




#616924 [Trường Xuân toán miền Nam]Vietnam TST 2016 Mock Test 1

Đã gửi bởi baopbc on 25-02-2016 - 21:13 trong Thi HSG cấp Tỉnh, Thành phố. Olympic 30-4. Đề thi và kiểm tra đội tuyển các cấp.

Chỗ đó em giải hơi rườm, thực ra nó là bài toán sau:

Cho tam giác $ABC$. $E,F$ lần lượt là hình chiếu từ $B,C$ xuống $AC,AB$. $M$ là trung điểm $AC$. $J_{E}$ là tâm bàng tiếp góc $E$. Chứng minh rằng: $\angle AJ_{E}P=\angle C$

Chứng minh: Điều này tương đương với chứng minh: $AEFJ_{E}$ là tứ giác nội tiếp

Ta có biến đổi góc: $\angle FJ_{E}E=\angle EMF/2=\angle A$ (đẳng thức cuối đúng do $FM$ là trung tuyến ứng với cạnh huyền trong tam giác vuông)




#616713 [Trường Xuân toán miền Nam]Vietnam TST 2016 Mock Test 1

Đã gửi bởi baopbc on 24-02-2016 - 17:42 trong Thi HSG cấp Tỉnh, Thành phố. Olympic 30-4. Đề thi và kiểm tra đội tuyển các cấp.

Lời giải bài hình:

Câu a) Gọi $X$ là giao của $MN$ với với $T$. Do $J$ là tâm đường tròn $Euler$ nên $J$ là trung điểm $OH$. $T'$ đối xứng với $T$ qua $J$ thì $HOTT'$ là hình bình hành $\Rightarrow T'O//HT \Rightarrow T'O$ vuông góc với $MN$ hay $T',O,X$ thẳng hàng. Vậy $X$ là trung điểm $MN$.

$\Rightarrow K$ thuộc $HX$. Mặt khác theo tính chất của đường tròn $Euler$, kết hợp với $XH=XK$ thì $K$ thuộc $(O)$.

Vậy $K$ luôn chạy trên một đường cố định.

Câu b)Gọi $M$ là trung điểm $AC$. $P,Q$ là chân đường cao hạ từ $B,C$ xuống $AC,AB$.

Áp dụng bổ đề $Sayawama$ mở rộng cho tâm bàng tiếp với $\triangle PMQ$ thì $XY$ đi qua tâm bàng tiếp $\angle P$.

$R$ là hình chiếu của $E$ lên $AB$. Dễ thấy $\triangle PRZ\sim \triangle BEI$ nên $Z$ thuộc phân giác $\angle MPQ$.

Gọi $J_{P}$ là tâm bàng tiếp $\angle P$ thì $J_{Q},Z,P$ thẳng hàng.

$AYXZ$ là hình thoi khi và chỉ khi $\triangle J_{Q}AZ$ cân. Ta có biến đổi góc: $\angle AZJ_{Q}=90^{\circ}-\angle C/2$

Vậy ta chỉ cần chứng minh $\angle AJ_{Q}P=\angle C$. Gọi $S$ là chân đường phân giác kẻ từ $B$ xuống $AC$. Dễ thấy $\triangle BSC\sim \triangle AJ_{Q}P(c.g.c)$

Vậy ta có điều phải chứng minh!

Hình gửi kèm

  • Vietnam TST Mockup cái 2.png
  • Vietnam TST Mockup cái 1.jpg



#615141 [Tổ hợp] THPT tháng 12: Tính tổng các số tribi

Đã gửi bởi baopbc on 15-02-2016 - 11:31 trong Thảo luận đề thi VMEO IV

Mình thành thật xin lỗi, minh đã gõ vào Word rồi nhưng không post lên được!

Mình sẽ gửi lên sau:

P/s: Anh Toàn cho em hỏi: $S_{n+1}=S_{n}+2^{n-1}$ có đúng không ạ!




#615145 [Tổ hợp] THPT tháng 12: Tính tổng các số tribi

Đã gửi bởi baopbc on 15-02-2016 - 12:27 trong Thảo luận đề thi VMEO IV

Lời giải của mình: Ta có:$S_{n+1}=\sum_{k=1}^{2^{n+1}}T(k)=\sum_{k=1}^{2^{n}}T(k)+\sum_{k=2^{n}+1}^{2^{n+1}}T(k)=S_{n}+\sum_{k=2^{n}+1}^{2^{n+1}}$

Do $T(2^{n+1})=0$ nên $\sum_{k=2^{n}+1}^{2^{n+1}}=\sum_{k=2^{n}+1}^{2^{n}+(2^{n}-1)}$

Ta xét các số có dạng: $2^{n}+k (1\leq k\leq 2^{n}-1)$

Trường hợp 1: $k<2^{n-1}\Rightarrow 2^{n}+k=1.2^{n}+0.2^{n-1}+k\Rightarrow T(2^{n}+k)=T(k)$(1)

Trường hợp 2: $k\geq 2^{n-1}\Rightarrow 2^{n}+k=1.2^{n}+1.2^{n-1}+k-2^{n-1}\Rightarrow T(2^{n}+k)=T(k)+1$(2)

Từ (1)(2): $\Rightarrow \sum_{k=2^{n}+1}^{2^{n}+(2^{n}-1)}=\sum_{k=1}^{2^{n}}+2^{n-1}=S_{n}+2^{n-1}$

$\Rightarrow S_{n+1}=2S_{n}+2^{n-1}$

Áp dụng công thức sai phân ta tìm được $S_{n}$.

P/s: Chỗ cuối cũng có thể giải thích như sau:

$S_{n+1}=2S_{n}+2^{n-1}=2^{2}S_{n-1}+2.2^{n-1}=...=2^{n}.S_{1}+n.2^{n-1}=n.2^{n-1}$




#615044 [Đại số] THCS tháng 12: Chứng minh là số hữu tỷ

Đã gửi bởi baopbc on 14-02-2016 - 20:40 trong Thảo luận đề thi VMEO IV

Em thấy bài này hơi vô lí! Với mọi $a,b,c$ thuộc $S$ thì $a^3+b^3+c^3-3abc$ là số hứu tỉ!

Nếu vậy thì biểu thức cần chứng minh phải phụ thuộc cả a,b,c chứ!

Giả dụ: $S=(\sqrt[3]{2};1;\sqrt[3]{4})$ thì $a^3+b^3+c^3-3abc$ là số hữu tỉ!

Nếu chọn $a=\sqrt[3]{2}$;$b=1$ thì $\frac{a-b}{a+b}=\frac{\sqrt[3]{2}-1}{\sqrt[3]{2}+1}=\frac{1}{(\sqrt[3]{2}+1)(\sqrt[3]{4}+\sqrt[3]{2}+1)}=\frac{1}{1+\frac{2}{\sqrt[3]{2}+1}}$ là số vô tỉ!

Ai có thể giải thích giùm em được không?




#615097 [Đại số] THCS tháng 12: Chứng minh là số hữu tỷ

Đã gửi bởi baopbc on 14-02-2016 - 22:34 trong Thảo luận đề thi VMEO IV

Có vẻ như em quên mất tính chất không nhất thiết phân biệt của $a,b,c$ rồi.  ;)

Vâng cảm ơn anh nhiều!

Lời giải: Trường hợp 1: $S$ có duy nhất một phần tử $a$ thì $a^{3}+b^{3}+c^{3}-3abc=3a^{3}-3a^{3}=0$ hữu tỉ và $\frac{a-b}{a+b}=0$ là số hữu tỉ.

Trường hợp 2: $S$ có nhiều hơn 1 phần tử. Xét a,b là hai giá trị bất kì thuộc $S$.Theo giả thiết với $(a,b,c)\sim (a,b,b)$;$(a,b,c)\sim (a,a,b)$ ta suy ra:

$a^{3}+2b^{3}-3b^{2}a$ và $b^{3}+2a^{3}-3a^{2}b$ hữu tỉ.

$\Rightarrow a^{3}+b^{3}-ab(a+b)=(a+b)(a-b)^{2}$ và $a^{3}-b^{3}+3ab(b-a)=(a-b)^{3}$ là số hữu tỉ.

Do thương của hai số hữu tỉ là một số hữu tỉ nên: $\frac{(a-b)^{3}}{(a+b)(a-b)^{2}}=\frac{a-b}{a+b}$ là số hữu tỉ.

Vậy ta có điều phải chứng minh./




#614571 [Đại số]THPT tháng 12: $a^2+b^2+c^2-ab-bc-ca \geqslant k\left|...

Đã gửi bởi baopbc on 12-02-2016 - 20:23 trong Thảo luận đề thi VMEO IV

Anh dogsteven làm vậy thì chết em rồi! :(

Em giải thế này: P/s: Sai ở đâu nhờ các anh tìm giúp:

2.1) Cho c=0, ta được$a^{2}+b^{2}-ab\geq k\left | \frac{ab(a-b)}{a+b} \right |$(*)

Không giảm tính tổng quát, giả sử a>b thì (*) tương đương với

$k\leq \frac{(a+b)(a^{2}+b^{2}-ab)}{ab(a-b)}=\frac{a^{3}+b^{3}}{ab(a-b)}$

Lại có:$\frac{a^{3}+b^{3}}{ab(a-b)}> \frac{a+b}{a-b}> 1$

Cho $a\rightarrow b$ và $b\rightarrow 0$ thì $\frac{a^{3}+b^{3}}{ab(a-b)}\rightarrow 1$

Vậy k lớn nhất bằng 1

Ta chứng minh k=1 là giá trị lớn nhất cần tìm tức là chứng minh:

$a^{2}+b^{2}+c^{2}-ab-bc-ac\geq \left | \sum \frac{a^{3}-b^{3}}{a+b} \right |=\left |\sum (a^{2}-b^{2}-\frac{ab(a-b)}{a+b} \right |=$$\left | \sum \frac{ab(a-b)}{a+b} \right |$

Trước tiên ta có đánh giá sau:$a(a-b)^{2}\geq 0\Leftrightarrow a^{3}+ab^{2}\geq 2a^{2}b\Leftrightarrow (a^{2}+ab)(a+b)\geq 4a^{2}b\Leftrightarrow \frac{a^{2}+ab}{2}\geq \frac{2a^{2}b}{a+b}$

Không mất tính tổng quát giả sử: $\sum \frac{ab(a-b)}{a+b}\geq 0$ thì:

Vế phải=$\sum \frac{ab(a-b)}{a+b}$=$\sum \frac{2a^{2}b}{a+b}-\sum ab\leq \sum \frac{a^{2}+ab}{2}-\sum ab=\frac{\sum a^{2}-\sum ab}{2}\leq \sum a^{2}-\sum ab$=vế trái

Vậy bất đẳng thức đúng $\Rightarrow$ k=1 là giá trị lớn nhất cần tìm




#614598 [Đại số]THPT tháng 12: $a^2+b^2+c^2-ab-bc-ca \geqslant k\left|...

Đã gửi bởi baopbc on 12-02-2016 - 21:41 trong Thảo luận đề thi VMEO IV

 

Tìm hằng số $k$ lớn nhất sao cho bất đẳng thức \[a^2+b^2+c^2-ab-bc-ca \geqslant k\left|\frac{a^3-b^3}{a+b}+\frac{b^3-c^3}{b+c}+\frac{c^3-a^3}{c+a}\right|\] luôn đúng với mọi số thực không âm $a, b, c$ thỏa mãn $(a+b)(b+c)(c+a)>0$.
Anh Nguyễn Văn Huyện

Bài này có lẽ được tổng quát từ bài: Moldova TST 2004

Bài đó như sau: Cho a,b,c là các số thực dương. Chứng minh rằng:

$\left | \sum \frac{a^{3}-b^{3}}{a+b} \right |\leq \frac{\sum (a-b)^{2}}{4}$

Xem chi tiết hơn tại đây: https://mathifc.wordpress.com/page/2/